¿Cómo resolver la "ecuación de onda EM" para el campo de carga en movimiento uniforme?

¿Es posible demostrar que el campo de una carga en movimiento uniforme , que está de acuerdo con la ley de Biot-Savart, está dado por...

mi ( r , t ) = k q ( 1 v 2 / C 2 ( 1 v 2 pecado 2 θ / C 2 ) 3 / 2 ) ( r ^ γ 2 ( X v t ) 2 + y 2 + z 2 )

... satisface la siguiente ecuación (lejos de la carga en sí)?

C 2 2 mi = 2 t 2 mi

Editar:

que el movimiento uniforme no produce ondas. También sé que esta ecuación es válida solo para espacios vacíos lejos de cargas y corrientes.

Pero de alguna manera esta ecuación todavía tiene que ser válida para el campo eléctrico "arrastrado" inercialmente después de que la carga se mueva con velocidad constante.

Es muy difícil para mí entender esta idea. Cualquier referencia sería muy bienvenida. Intenté resolverlo directamente pero fallé.

Debe editar su publicación para dejar en claro que sabe que una carga en movimiento uniforme no genera una onda, sin embargo, los campos aparentemente tienen que satisfacer esta ecuación. No sé la respuesta (pregunta de votación positiva), pero observo que no hay un término fuente en su ecuación de onda... tal vez eso marque la diferencia.
Un enfoque para esto sería ir primero al marco de reposo de la carga, en este marco el campo eléctrico es particularmente simple, a saber mi = q 4 π ϵ 0 r 2 , donde es sencillo mostrar (ya que no hay derivadas temporales) y luego transformar a un marco donde la carga se mueve y observar que 2 t 2 + 1 C 2 2 es invariante, ver por ejemplo Purcell, Electricity and Magnetism .
Hola @jim, ¿es correcto decir que la llamada "ecuación de onda EM" obviamente tiene soluciones no ondulatorias (como un campo en movimiento inercial)? ¿Por qué se llama ecuación de onda después de todo?
@VitalyKorzhik Creo que debido a que la ecuación EM puede admitir soluciones de onda, se la conoce como ecuación de onda EM . No se hace ninguna distinción particular cuando observa ecuaciones independientes del tiempo. Además, puede tener soluciones dependientes del tiempo que no tienen que ser sinusoidales, que la gente suele considerar como una onda.
@jim ¿Hay alguna razón obvia por la que la función de campo no satisfaga v 2 2 mi = 2 t 2 mi (con v en lugar de C )? Porque no puede satisfacer ambos al mismo tiempo. Esto probablemente tenga algo que ver con la transformación de Lorentz...
Sí, la transformación de Lorentz significa que solo cuando v = C es la ecuación de onda invariante. Otra forma de pensar en esto es para v < C sería posible ir a un marco donde v = 0 lo que significa que podría aumentar a un marco donde las ondas em serían estáticas. La siguiente pregunta y respuesta pueden ser de su interés: physics.stackexchange.com/questions/248499/…

Respuestas (1)

La gente no está entendiendo tu pregunta. Creo que quiere que alguien verifique explícitamente que los campos producidos en su caso especial obedecen (o no) la ecuación de onda generalmente válida . Después de todo, el campo producido no parece una onda. Una solución general de la ecuación de onda para una perturbación que viaja en la X la direccion es mi ( r , t ) = mi 0 ( X C t , y , z ) . entonces supongo

mi ( r , t ) = 1 4 π ϵ 0 q [ ( X C t ) 2 + y 2 + z 2 ] r ^
(dónde r ^ puntos alejados de la carga en cualquier instante) es una solución de la ecuación de onda.

Obviamente, esto es bueno solo para velocidades no relativistas. Y mi notación para el vector unitario no es muy buena... aunque espero que sirva de algo.

Comentario

Veo que ha editado su pregunta para incluir una expresión explícita de la expresión relativista para el campo. Creo que está claro que lo que hice para el caso no relativista funciona igual de bien para el caso relativista.

Eliminé este comentario y lo agregué a mi respuesta.
He agregado la dependencia del tiempo explícitamente de acuerdo con sus sugerencias, espero que se vea bien. Supongo que la parte relativista puede ignorarse por simplicidad @Mathaholic
Es casi correcto... tu no defines r ^ , y yo tampoco. Con una definición correcta de r ^ ¡Creo que lo que escribiste es manifiestamente una solución a la ecuación de onda!
El problema es que tengo v en lugar de C en mi ecuación, así que no veo cómo resuelve la ecuación de Maxwell con C en eso. ¿Puedes elaborar un poco?
Ajá. Sabía que parecía demasiado fácil. :)